Prove function is differentiable at a point











up vote
0
down vote

favorite












Prove that the function
$$f(x) = left{
begin{array}{ll}
x^2, & x in mathbb{Q} \
0, & x in mathbb{Q}^c \
end{array}
right.$$



is differentiable at $x = 0.$



I'm not sure how to calculate the limits for this function (the density of $mathbb{Q}$ in $mathbb{R}$ is confusing me). Any advice would be appreciated!










share|cite|improve this question









New contributor




51n84d is a new contributor to this site. Take care in asking for clarification, commenting, and answering.
Check out our Code of Conduct.
















  • 2




    Apply the squeeze lemma on the differential quotient. Find two functions it is always sandwiched between, which have limit zero.
    – астон вілла олоф мэллбэрг
    Nov 13 at 10:05












  • Here's a very similar question (just subtract $x$): math.stackexchange.com/questions/298482/…
    – Hans Lundmark
    Nov 13 at 11:03















up vote
0
down vote

favorite












Prove that the function
$$f(x) = left{
begin{array}{ll}
x^2, & x in mathbb{Q} \
0, & x in mathbb{Q}^c \
end{array}
right.$$



is differentiable at $x = 0.$



I'm not sure how to calculate the limits for this function (the density of $mathbb{Q}$ in $mathbb{R}$ is confusing me). Any advice would be appreciated!










share|cite|improve this question









New contributor




51n84d is a new contributor to this site. Take care in asking for clarification, commenting, and answering.
Check out our Code of Conduct.
















  • 2




    Apply the squeeze lemma on the differential quotient. Find two functions it is always sandwiched between, which have limit zero.
    – астон вілла олоф мэллбэрг
    Nov 13 at 10:05












  • Here's a very similar question (just subtract $x$): math.stackexchange.com/questions/298482/…
    – Hans Lundmark
    Nov 13 at 11:03













up vote
0
down vote

favorite









up vote
0
down vote

favorite











Prove that the function
$$f(x) = left{
begin{array}{ll}
x^2, & x in mathbb{Q} \
0, & x in mathbb{Q}^c \
end{array}
right.$$



is differentiable at $x = 0.$



I'm not sure how to calculate the limits for this function (the density of $mathbb{Q}$ in $mathbb{R}$ is confusing me). Any advice would be appreciated!










share|cite|improve this question









New contributor




51n84d is a new contributor to this site. Take care in asking for clarification, commenting, and answering.
Check out our Code of Conduct.











Prove that the function
$$f(x) = left{
begin{array}{ll}
x^2, & x in mathbb{Q} \
0, & x in mathbb{Q}^c \
end{array}
right.$$



is differentiable at $x = 0.$



I'm not sure how to calculate the limits for this function (the density of $mathbb{Q}$ in $mathbb{R}$ is confusing me). Any advice would be appreciated!







limits derivatives continuity






share|cite|improve this question









New contributor




51n84d is a new contributor to this site. Take care in asking for clarification, commenting, and answering.
Check out our Code of Conduct.











share|cite|improve this question









New contributor




51n84d is a new contributor to this site. Take care in asking for clarification, commenting, and answering.
Check out our Code of Conduct.









share|cite|improve this question




share|cite|improve this question








edited Nov 13 at 10:57









Sam Streeter

1,422417




1,422417






New contributor




51n84d is a new contributor to this site. Take care in asking for clarification, commenting, and answering.
Check out our Code of Conduct.









asked Nov 13 at 10:03









51n84d

273




273




New contributor




51n84d is a new contributor to this site. Take care in asking for clarification, commenting, and answering.
Check out our Code of Conduct.





New contributor





51n84d is a new contributor to this site. Take care in asking for clarification, commenting, and answering.
Check out our Code of Conduct.






51n84d is a new contributor to this site. Take care in asking for clarification, commenting, and answering.
Check out our Code of Conduct.








  • 2




    Apply the squeeze lemma on the differential quotient. Find two functions it is always sandwiched between, which have limit zero.
    – астон вілла олоф мэллбэрг
    Nov 13 at 10:05












  • Here's a very similar question (just subtract $x$): math.stackexchange.com/questions/298482/…
    – Hans Lundmark
    Nov 13 at 11:03














  • 2




    Apply the squeeze lemma on the differential quotient. Find two functions it is always sandwiched between, which have limit zero.
    – астон вілла олоф мэллбэрг
    Nov 13 at 10:05












  • Here's a very similar question (just subtract $x$): math.stackexchange.com/questions/298482/…
    – Hans Lundmark
    Nov 13 at 11:03








2




2




Apply the squeeze lemma on the differential quotient. Find two functions it is always sandwiched between, which have limit zero.
– астон вілла олоф мэллбэрг
Nov 13 at 10:05






Apply the squeeze lemma on the differential quotient. Find two functions it is always sandwiched between, which have limit zero.
– астон вілла олоф мэллбэрг
Nov 13 at 10:05














Here's a very similar question (just subtract $x$): math.stackexchange.com/questions/298482/…
– Hans Lundmark
Nov 13 at 11:03




Here's a very similar question (just subtract $x$): math.stackexchange.com/questions/298482/…
– Hans Lundmark
Nov 13 at 11:03










2 Answers
2






active

oldest

votes

















up vote
1
down vote



accepted










We have
$$lim_{xto 0,xin Bbb {Q}}dfrac{f(x)-f(0)}{x-0}=0$$
and
$$lim_{xto 0,xin mathbb{R}setminusmathbb{Q}}dfrac{f(x)-f(0)}{x-0}=0.$$
Hence, $f$ is differentiable at $x=0$.






share|cite|improve this answer




























    up vote
    0
    down vote













    We need that



    $$limlimits_{x to 0} frac{f(x)}{x} = 0$$ (having already done some simplification, and noting that the derivative must be zero if it exists, since the function is even).



    Now, $$frac{f(x)}{x} = left{array{x,&x in mathbb{Q}\0,&x in mathbb{Q}^c}right.$$



    Clearly, both the functions $g(x) = x$ and $h(x) = 0$ have the limit $0$ at $0$, so by what I guess you might have called the subset limit theorem, so does $frac{f(x)}{x}$, so the limit is 0. In particular, the limit exists, so $f$ is differentiable at $0$.






    share|cite|improve this answer





















      Your Answer





      StackExchange.ifUsing("editor", function () {
      return StackExchange.using("mathjaxEditing", function () {
      StackExchange.MarkdownEditor.creationCallbacks.add(function (editor, postfix) {
      StackExchange.mathjaxEditing.prepareWmdForMathJax(editor, postfix, [["$", "$"], ["\\(","\\)"]]);
      });
      });
      }, "mathjax-editing");

      StackExchange.ready(function() {
      var channelOptions = {
      tags: "".split(" "),
      id: "69"
      };
      initTagRenderer("".split(" "), "".split(" "), channelOptions);

      StackExchange.using("externalEditor", function() {
      // Have to fire editor after snippets, if snippets enabled
      if (StackExchange.settings.snippets.snippetsEnabled) {
      StackExchange.using("snippets", function() {
      createEditor();
      });
      }
      else {
      createEditor();
      }
      });

      function createEditor() {
      StackExchange.prepareEditor({
      heartbeatType: 'answer',
      convertImagesToLinks: true,
      noModals: true,
      showLowRepImageUploadWarning: true,
      reputationToPostImages: 10,
      bindNavPrevention: true,
      postfix: "",
      imageUploader: {
      brandingHtml: "Powered by u003ca class="icon-imgur-white" href="https://imgur.com/"u003eu003c/au003e",
      contentPolicyHtml: "User contributions licensed under u003ca href="https://creativecommons.org/licenses/by-sa/3.0/"u003ecc by-sa 3.0 with attribution requiredu003c/au003e u003ca href="https://stackoverflow.com/legal/content-policy"u003e(content policy)u003c/au003e",
      allowUrls: true
      },
      noCode: true, onDemand: true,
      discardSelector: ".discard-answer"
      ,immediatelyShowMarkdownHelp:true
      });


      }
      });






      51n84d is a new contributor. Be nice, and check out our Code of Conduct.










       

      draft saved


      draft discarded


















      StackExchange.ready(
      function () {
      StackExchange.openid.initPostLogin('.new-post-login', 'https%3a%2f%2fmath.stackexchange.com%2fquestions%2f2996554%2fprove-function-is-differentiable-at-a-point%23new-answer', 'question_page');
      }
      );

      Post as a guest















      Required, but never shown

























      2 Answers
      2






      active

      oldest

      votes








      2 Answers
      2






      active

      oldest

      votes









      active

      oldest

      votes






      active

      oldest

      votes








      up vote
      1
      down vote



      accepted










      We have
      $$lim_{xto 0,xin Bbb {Q}}dfrac{f(x)-f(0)}{x-0}=0$$
      and
      $$lim_{xto 0,xin mathbb{R}setminusmathbb{Q}}dfrac{f(x)-f(0)}{x-0}=0.$$
      Hence, $f$ is differentiable at $x=0$.






      share|cite|improve this answer

























        up vote
        1
        down vote



        accepted










        We have
        $$lim_{xto 0,xin Bbb {Q}}dfrac{f(x)-f(0)}{x-0}=0$$
        and
        $$lim_{xto 0,xin mathbb{R}setminusmathbb{Q}}dfrac{f(x)-f(0)}{x-0}=0.$$
        Hence, $f$ is differentiable at $x=0$.






        share|cite|improve this answer























          up vote
          1
          down vote



          accepted







          up vote
          1
          down vote



          accepted






          We have
          $$lim_{xto 0,xin Bbb {Q}}dfrac{f(x)-f(0)}{x-0}=0$$
          and
          $$lim_{xto 0,xin mathbb{R}setminusmathbb{Q}}dfrac{f(x)-f(0)}{x-0}=0.$$
          Hence, $f$ is differentiable at $x=0$.






          share|cite|improve this answer












          We have
          $$lim_{xto 0,xin Bbb {Q}}dfrac{f(x)-f(0)}{x-0}=0$$
          and
          $$lim_{xto 0,xin mathbb{R}setminusmathbb{Q}}dfrac{f(x)-f(0)}{x-0}=0.$$
          Hence, $f$ is differentiable at $x=0$.







          share|cite|improve this answer












          share|cite|improve this answer



          share|cite|improve this answer










          answered Nov 13 at 11:04









          1ENİGMA1

          938316




          938316






















              up vote
              0
              down vote













              We need that



              $$limlimits_{x to 0} frac{f(x)}{x} = 0$$ (having already done some simplification, and noting that the derivative must be zero if it exists, since the function is even).



              Now, $$frac{f(x)}{x} = left{array{x,&x in mathbb{Q}\0,&x in mathbb{Q}^c}right.$$



              Clearly, both the functions $g(x) = x$ and $h(x) = 0$ have the limit $0$ at $0$, so by what I guess you might have called the subset limit theorem, so does $frac{f(x)}{x}$, so the limit is 0. In particular, the limit exists, so $f$ is differentiable at $0$.






              share|cite|improve this answer

























                up vote
                0
                down vote













                We need that



                $$limlimits_{x to 0} frac{f(x)}{x} = 0$$ (having already done some simplification, and noting that the derivative must be zero if it exists, since the function is even).



                Now, $$frac{f(x)}{x} = left{array{x,&x in mathbb{Q}\0,&x in mathbb{Q}^c}right.$$



                Clearly, both the functions $g(x) = x$ and $h(x) = 0$ have the limit $0$ at $0$, so by what I guess you might have called the subset limit theorem, so does $frac{f(x)}{x}$, so the limit is 0. In particular, the limit exists, so $f$ is differentiable at $0$.






                share|cite|improve this answer























                  up vote
                  0
                  down vote










                  up vote
                  0
                  down vote









                  We need that



                  $$limlimits_{x to 0} frac{f(x)}{x} = 0$$ (having already done some simplification, and noting that the derivative must be zero if it exists, since the function is even).



                  Now, $$frac{f(x)}{x} = left{array{x,&x in mathbb{Q}\0,&x in mathbb{Q}^c}right.$$



                  Clearly, both the functions $g(x) = x$ and $h(x) = 0$ have the limit $0$ at $0$, so by what I guess you might have called the subset limit theorem, so does $frac{f(x)}{x}$, so the limit is 0. In particular, the limit exists, so $f$ is differentiable at $0$.






                  share|cite|improve this answer












                  We need that



                  $$limlimits_{x to 0} frac{f(x)}{x} = 0$$ (having already done some simplification, and noting that the derivative must be zero if it exists, since the function is even).



                  Now, $$frac{f(x)}{x} = left{array{x,&x in mathbb{Q}\0,&x in mathbb{Q}^c}right.$$



                  Clearly, both the functions $g(x) = x$ and $h(x) = 0$ have the limit $0$ at $0$, so by what I guess you might have called the subset limit theorem, so does $frac{f(x)}{x}$, so the limit is 0. In particular, the limit exists, so $f$ is differentiable at $0$.







                  share|cite|improve this answer












                  share|cite|improve this answer



                  share|cite|improve this answer










                  answered Nov 13 at 10:15









                  user3482749

                  64939




                  64939






















                      51n84d is a new contributor. Be nice, and check out our Code of Conduct.










                       

                      draft saved


                      draft discarded


















                      51n84d is a new contributor. Be nice, and check out our Code of Conduct.













                      51n84d is a new contributor. Be nice, and check out our Code of Conduct.












                      51n84d is a new contributor. Be nice, and check out our Code of Conduct.















                       


                      draft saved


                      draft discarded














                      StackExchange.ready(
                      function () {
                      StackExchange.openid.initPostLogin('.new-post-login', 'https%3a%2f%2fmath.stackexchange.com%2fquestions%2f2996554%2fprove-function-is-differentiable-at-a-point%23new-answer', 'question_page');
                      }
                      );

                      Post as a guest















                      Required, but never shown





















































                      Required, but never shown














                      Required, but never shown












                      Required, but never shown







                      Required, but never shown

































                      Required, but never shown














                      Required, but never shown












                      Required, but never shown







                      Required, but never shown







                      Popular posts from this blog

                      How do I know what Microsoft account the skydrive app is syncing to?

                      When does type information flow backwards in C++?

                      Grease: Live!